Thursday, February 21, 2013

Success of the modified Lorentz perpendicular action experiment

29 comments:

  1. Very difficult to say anything since your wound up coil isn't perfectly symmetrical.
    (btw, is that wood's metal, mercury, or something else?)
    What do I expect when the current turns on - that the coil gets pulled or pushed away from the magnet (assuming that the magnet is arranged north/south vertically). If your axis of rotation isn't close enough to the perfect center, there will be an imbalance of forces and a torque.
    In the ideal case, the electromagnet should only feel a pull or a push, but no torque. Try to place a strong but light neodymium magnet in the middle of your coil but don't run the current. The magnet will sort of simulate the field when the current is on. I'd bet that the coil would rotate the same way as it does now.

    ReplyDelete
    Replies
    1. It is mercury for support and connexion.

      The point is that the torque is different when the axis of rotation turns 90°. For classical theory, the torque has the same value when the axis is 0° or 90°.

      Delete
    2. ahh, I wrote this whole thing here but blogspot just refreshed the page when I pressed publish :(
      long story short - in the first case there is an imbalance of force due to a non-perfect axis. In the second case, the axis of rotating limits how any possible force from the top magnet would affect the coil.
      https://awwapp.com/s/4a/f2/a9.png

      Delete
    3. unless you mean some other situation, or I misunderstood your set up geometry, I don't see the problem here for the classical theory.

      Delete
    4. The thing is that the coil rotates when it faces the magnet, but it not rotates when it is perpendicular to the magnet. As the B field in the z direction is the same, it should generate the same Lorentz force.

      Delete
    5. Wow, I put way too much work into this problem, and I just realized a more philosophical answer to this.
      http://i.imgur.com/s5PgyB2.jpg

      In the first case, the torque is caused by an off-center axis which results in a large lever arm against a force of some magnitude.

      In the second case, there is also torque, but because part of the coil wants to move one way, while another part wants to move another way, the part left over to multiply against the large lever arm is much smaller.

      In any case where there is an equal force throughout see-saw like object, an off-center axis would result in the following torque:
      of center = x. On one side, you have force along a length of L/2-x. On the other side, this gets canceled out, but there is an extra 2x distance at the very edge of the see-saw which results in a torque of about (F/L)*2x*(L/2+x).

      This is why there is a torque at all. In the second case, the net F is much smaller. I am almost finished with an excel macro sheet that you can input any arrangement of magnets and coils and it will tell you the force on every part of every coil. So, if this wordy argument isn't sufficient, I can get you the numbers in a day or so).


      In more philosophical terms: magnets always move towards a position of less potential energy. That's why perpetual motion can't be done using magnets, because any motion will get you to a lower or even potential energy. In the first case, the coil wants to pull towards the coil, so it's pretty straight forward.
      In the second case, the much larger force is the torque that tries to rotate the coil to line up with the magnetic field. However, the axis you've limited it to is the exact axis about which rotation doesn't improve its position with the magnet.

      Point is - if the geometry was ideal, there would be no torque and rotation in either case.

      What is your timezone? I'd much rather chat than talk over long forum delays. It's much faster to get thoughts across.

      I'll probably finish the excel macro so let me know if you'd like to use it to test out forces in arrangements.

      Delete
    6. I'm in France.

      Excel is not good enough. I use Matlab that is much more precise and rapid so that you can set thousand of nods for numerical calculation.

      Delete
    7. Have you read my explanation of the perpendicular action? Here
      Lorentz perpendicular action experiment and Lorentz force law
      http://pengkuanem.blogspot.fr/2013/02/lorentz-perpendicular-action-experiment.html

      In this article I have explained most of your philosophical thinking shown in your image.

      Delete
    8. Is the coil not centered along the magnetic field? It is below?
      Can you give me some numbers as to the dimensions of the magnet, dimensions of the coil, location of the axis with respect to the magnet...etc. - so that I may completely replicate the situation in my macro.

      Matlab is much better than excel - but excel let's me play around much more easily. Regarding precision - we don't need to be precise to the 5th decimal place to know the outcome of a situation.

      Looking forward to hearing from you on the dimensions of your experiment. (I was under the impression that the coil axis and coil are centered along face of the magnet (but at a distance))

      Delete
    9. Also, no I haven't read your other paper - as you have a lot and I didn't have time to read them all :)
      I'm looking at it, but I need to know in more details the dimensions to understand exactly where the magnetic field is pointing and with what relative magnitude at each position.

      Delete
    10. I think your diagram of the vector of the magnetic field isn't correct - from what I can gather from your paper and the video. If you give me the exact dimensions, I'll give you the exact numbers.

      Delete
    11. The magnet is an ordinary ferrite magnet of dimension 60*18*28 mm. The direction of the poles, the position of the coil with respect to the magnet are shown in the second article
      Lorentz perpendicular action experiment and Lorentz force law
      http://pengkuanem.blogspot.fr/2013/02/lorentz-perpendicular-action-experiment.html
      The coil has 15 turns, is 10 mm above the table, at about 150-160 mm from the magnet. The current is about 1A. The magnet is put directly on the table.

      The coil is not attracted to the magnet as a whole as shown in the video. Since the support of the coil is not strong, any attraction would be seen.

      Delete
    12. That was a lot of work. The result: not quite sure:
      http://i.imgur.com/zvZlRAa.jpg
      First I modeled a circular coil, afterwards a square coil.
      In the cases where the coil is facing -Y direction the magnet and it is centered, there is no torque on it.
      In the cases where the coil is facing -Y direction, but is lowered, like in your set up, the shape of the graph shows an obvious net torque.
      In the cases where the coil is facing the +x direction, the graphs show a general even behavior (no torque), but there ends up to be a small torque anyway.
      I looked at your video, and I do see that the coil tries to twist (in the opposite direction as when it is facing, as my macro shows), but seems to get stuck.

      A lot of things could be inconsistent - anywhere from how i modeled the ferro-magnet (as 2 rectangular loops of current), to the exact shape of the coil.
      I'll try to build something like this and make a video. I won't use liquid metal, but as I will be using strong magnets, it shouldn't be a problem.

      I'd love to see your apparatus send current one way and the other, to demonstrate that it isn't a fault in the geometry. (that's what i'm going to try to do).

      Delete
    13. Just used string (axis) and magnets, and there is def a torque in both cases when you are below center. I think your set up is incorrect (since the coil of wire behaves exactly as a disc magnet).

      Will post video later.

      Delete
    14. oops..and now 2 magnets cracked...they are so brittle :( ...should have used a cloth/paper barrier

      Delete
    15. This doesn't match your geometry, and because the magnets are very thin, the shape of the field is different, but here is the behavior that you should see
      https://www.youtube.com/watch?v=aw-02ss3bTo

      If you don't see this in your set up assuming similar magnetic fields there are only 2 options:

      Either you think that an electromagnet behaves differently than a magnet of the same strength in the same magnetic field
      or
      something is wrong in your set up (geometry/something getting stuck..etc

      thoughts?

      Delete
    16. Excellent work of computing and experiment!

      For me magnet and coil behave the same way. I think a coil of 1 000 amperes would be equivalent to a ferrite magnet of 10 mm thickness.

      In your data, I do not see why the 2 troughs of torque curve for the N-y direction coil do not have the same magnitude. The curve should be like a sinus.

      Now that you can get theoretical and experimental values, you are able to check the validity of the theory, that is, to compare theoretical and experimental results. What I’m claiming is that these results, the torques, are different. But the difference is difficult to show, because electromagnetic theory is a very strong theory, which is correct in ordinary conditions. This is why you find that your test magnet rotates accordingly to the theory. The object of this experiment is not to show that the theory is correct, but it is wrong. And it is wrong only in some specific points.

      In order to show the difference said above, I have used a long rectangular magnet, a small coil and placed them in a particular position so that this difference is apparent, a specific point. The point 155 mm from the field magnet (which makes the field to be tested) and 10 mm above the base plane is such a point. Here the torque on the coil in N-x direction is much smaller than that for N-y direction.

      When testing the N-x torque with your magnetic setup, you should not turn the test magnet around the field magnet but rotate it like my coil so that you can compare the torque for N-x and N-y direction for the same point, (x=0,y=0,z variable). Slide it vertically to find the point where the difference of torque is maximal. The torque N-x is small but not zero, that for N-y direction is greater. This is the difference I tried to show.

      In your video, I see that the field magnet is a small square magnet. The space between the center and the bottom is too small to see the variation of the torque along the z axis and the field of a rectangular magnet is very different from that of a square magnet. Also, your test magnet is too big, even bigger than the field magnet. It is too big to measure the field of regions small enough to consider the field uniform. A magnet of 2 mm will be better. My coil is 10 mm of diameter.

      For my reversing the current experiment, I cannot do it right now because I have already scrapped my setup to make place for other experiments. But I will do it in the future.

      Delete
    17. I'm assuming you mean the second graph down in the left column. I think it is because of the direction of the magnetic field for the lower part of the coil. The second from the bottom in the left column is the same situation but with a square coil. The lower a horizontal wire is compared to the center of the magnet, the move vertical the magnetic field lines tend to be - and the more vertical they are, the more force is in the y-axis direction.

      The torques between the x and y direction set up are mostly different due to the geometry. In the x direction set up, my macro says that it will be a little less - but again this is hard to replicate in the program since the magnet you use isn't perfect, so there could be a lot of slight variations in the field.

      In my video, i just stacked a bunch of neodymium magnets as the field source, and 2 pinching a string as the receiver - just to show that there is a torque in both cases.

      In my set up, the center of the coil is the same for the N+x and N-y direction. I expected the torques to be much more different, but with the parameters that I used, the torques were about the same. The torques increase as you get further from the center.

      Imagine you had a uniform vertical magnetic field. In that case, the magnetic field going through the coil would be identical, and the geometry would be identical, and rotating the axis about itself but keeping it perpendicular to the magnetic field would result in the same torque in both cases. This would be much easier to test just using some iron or large coils.

      In your case, it's a lot more finicky because you are dealing with a magnetic field whose angle changes throughout the test coil. You can make the test coil very small, and very far away, but that would result in a smaller force overall.

      I have a lot more graphs, and numbers in my excel sheet. If you want, you can provide some wire geometry, a volume, and a resolution, and I can send you a datasheet with B=b(x,y,z) for each point so that you can plot in matlab...as matlab is muuch better at plotting. Magnetic fields are difficult to plot well unless you have a 3d model that you can freely rotate (matlab can rotate 3d graphs..very convenient).

      Delete

    18. I suppose the curves of torques are drawn with respect to angle. That way, after 180°, you should have the opposite torque with the same magnitude. Maybe the computed coil’s dimension is too big to have a uniform field in the occupied region. In a uniform field, the torque must be perfect sinus, and is identical for N-x and N-y.

      Delete
    19. if there is a net torque, then it wouldn't be a sin wave. The force would be like a sin wave, but a negative force on the opposite side of the lever results in the same positive torque.
      The lever arm was calculated as (Z of point on wire) - (Z of axis)
      The torque in one case was then F_x*lever arm or F_y*lever arm
      ...depending on the direction of the coil.

      Delete
    20. It is not a sin curve, but must be periodic. At angle a, torque=s, at angle a+180, torque=-s.

      Delete
    21. that isn't how torque is calculated. You pick an axis, and direction to call positive. So torque isn't really + or -, it's clockwise or counter-clockwise.
      This is why torque can't flip signs half way if you expect any net torque.

      Imagine a stick attached to a center point facing you like the hour and minute hands of a clock at 6:00 am.
      If the force on the top is rightwards, and you call that a positive force, and the force on the bottom is leftwards, and you call that a negative force - you'll see that the torque is clockwise in both directions - hence it doesn't change sign.
      If on the other hand, the force was uniform, then the torque from the top end would be one value while the torque at the bottom end would be the opposite value, and the net torque would be 0.
      0 net torque means 0 angular acceleration.

      Delete
    22. I think we are not talking about the same thing. Here is what I mean by "At angle a, torque=s, at angle a+180, torque=-s".
      http://4.bp.blogspot.com/-1tJz4IL6FuA/VGYOSNh__KI/AAAAAAAAAVw/nAwgke-vr5k/s1600/Capture.JPG

      When the coil tilts +180° more, the coil flips upside down. It is like the same coil with reversed current. So the force are opposite all over and the torque is reversed.

      Delete
    23. HAHA we are totally not talking about the same thing. My mistake..that's what I get for not labeling graph appropriately. The x-axis of the graph relates to a point on the coil. That is why for the round coils, the graphs are curvy, while for the square coil, they are very sharp.
      The angles of the coils haven't been altered in this per graph - the only angle change is changing the direction of the North-Face of the coil from [away from the field magnet, negative Y direction], or [right angle to the field magnet, positive X direction].
      Where X,Y,Z are as your video shows.

      Delete
    24. I understand better now. For me, it is the resultant torque that is important.

      Delete
  2. The point about the set up that I'm trying to make is that if you think a formula/law is wrong, you should be able to show it using a simple set up - not something complex. In your case, you are dealing with coils in a magnetic field and that is very easy to set up uniform fields to test out forces in all directions.

    I'm trying to do an experiment regarding a rail-gun..and that is tough because good railguns use very high current. So I went to the store and got 20 free disposable cameras to make a capacitor bank :)

    ReplyDelete
  3. I do not use uniform field because the theory is correct in this case. The incorrectness shows itself only in special points. To say clearly, magnetic field is vector field for classical theory and it is considered as the sum of horizontal and vertical component at any point. This is why classical theory predicts same torque for N-x and N-y direction. But in reality it is not. I have shown this in other articles of my blog. Because it is not a vector, it makes different torque in x and y direction.

    Can you compute for smaller coil that has same the torque in x and y direction?

    Stores in your country give camera for free?

    ReplyDelete
    Replies
    1. I have been doing this instead of work :( for a while, so I should slow down till the weekend. I will try different configurations later.

      In my country, and probably yours as well, there are tourist disposable cameras that are just returned to the store for processing. The stores here take out the film to develop the photos, but they either throw out or recycle the cameras themselves. The cameras are super cheap for them. I just called a store and asked if I can have a bunch that they are throwing out anyway, and they were happy to oblige. So now I have 20 140 or so microfarad capacitors and circuits to charge them to a voltage of about 300 V. (BTW, that is more than enough to pass through skin..i learned that the hard way and burned my finger).

      Delete
    2. Good to know about the camera. You learned me something.

      I see your enthusiasm. Work first of cause.

      Delete